Let$$2^{\ell-1}<p_1<p_2<\dots<p_t<2^\ell$$$$2^{\ell'-1}<q_1<q_2<\dots<q_m<2^{\ell'}$$be primes on the condition$$\phi(p_i)=2q_1^{a_1}\dots q_m^{a_m}$$($\phi$ is Euler Totient function) where$q_i$ are odd primes and$$a_1,a_2,\dots,a_m\in\mathbb N_{>0}$$ are integers. What is the minimum$\ell,\ell'\in\mathbb N$ for a given$m,t,t'\in\mathbb N$ necessary such that there is an integer$g\in[2,p_1]$ satisfying the property
$$g^{q_1^{b_1}\dots q_m^{b_m}}\equiv1\bmod p_i$$$$1\leq b_1+\dots+b_m\leq t'$$$$\forall i\in\{1,\dots,m\}\mbox{ }0\leq b_i\leq a_i?$$
Is there heuristics or is the problem studied?
- $\begingroup$What is that function $\lambda$?$\endgroup$ajotatxe– ajotatxe2025-09-04 21:27:58 +00:00CommentedSep 4 at 21:27
- $\begingroup$en.wikipedia.org/wiki/Carmichael_function it is same as totient for primes.$\endgroup$Turbo– Turbo2025-09-04 21:29:03 +00:00CommentedSep 4 at 21:29
- $\begingroup$Renamed to Totient.$\endgroup$Turbo– Turbo2025-09-04 21:32:37 +00:00CommentedSep 4 at 21:32
- $\begingroup$Not following. $\varphi(p_i)=p_i-1$ is even, so how could the $q_j$ all be odd?$\endgroup$lulu– lulu2025-09-04 22:10:00 +00:00CommentedSep 4 at 22:10
- $\begingroup$Good catch has to be multiplied by $2$.$\endgroup$Turbo– Turbo2025-09-04 22:12:52 +00:00CommentedSep 4 at 22:12
You mustlog in to answer this question.
Explore related questions
See similar questions with these tags.